Funciones continuas de NN\Bbb{N} a NN\Bbb{N} en la topología "co-pequeña"

En una publicación relacionada, pregunté sobre la topología "co-pequeña" en norte . Una de las preguntas era sobre la caracterización de las funciones continuas a partir de norte a sí mismo en esta topología. Algunos ejemplos de funciones continuas incluyen F ( norte ) = a norte + b , F ( norte ) = norte pag para 0 < pag 1 , la función de conteo primo F ( norte ) = π ( norte ) ; algunas funciones que no son continuas serían F ( norte ) = registro 2 ( norte ) + 1 , F ( norte ) = pag norte (el norte th prima), F ( norte ) = mi en norte .

Otros usuarios han dado resultados parciales. Ben muestra que si F : norte norte es continuo y F ( A ) es pequeño para cualquier conjunto grande A , entonces F es constante Greg Martin muestra que si F : norte norte satisface límite norte F ( norte ) / norte = en cualquier conjunto grande A , entonces F debe mapear algún gran subconjunto de A a un conjunto pequeño, y por lo tanto no puede ser continuo en esta topología. Creo que estoy preparado para dar una caracterización de qué tan rápido o lento es una función continua no constante F : norte norte puede crecer:

(Propuesto) Teorema. Una función F de norte a sí mismo es continuo en la topología co-pequeña iff solo si existen constantes positivas METRO , pag tal que:

  1. F ( norte ) METRO norte para todos menos un pequeño conjunto de enteros positivos norte ;

  2. F ( norte ) norte pag para todos menos un pequeño conjunto de enteros positivos norte .

La respuesta de Greg Martin se despacha de (1), y la respuesta de Ben implica que F ( norte ) excepto posiblemente en un pequeño conjunto de enteros positivos norte (los conjuntos finitos son pequeños, por lo que la preimagen de cualquier conjunto finito bajo una función continua no puede ser grande). Mi respuesta para la última parte se basa en pensar, por ejemplo, F ( norte ) = norte contra F ( norte ) = registro 2 ( norte ) + 1 .

Para F ( norte ) = norte , para cualquier entero k ,

norte F 1 ( k ) 1 norte 2 k ,
y así la preimagen de cualquier conjunto grande también es grande. Esto se generaliza a cualquier norte pag con 0 < pag 1 como en este caso norte F 1 ( k ) 1 norte 1 pag k .

OTO, por F ( norte ) = registro 2 ( norte ) + 1 , para cualquier entero k ,

norte F 1 ( k ) 1 norte 1 2 ,
y así cualquier conjunto infinito (incluso uno pequeño) tiene una gran preimagen.

Pregunta: ¿Es cierto que para cualquier F : norte norte , si 1 / k = o ( norte F 1 ( k ) 1 norte ) en un gran conjunto de enteros positivos k , eso F asigna un conjunto grande a un conjunto pequeño? Y, si esto no es equivalente a (2) anterior, ¿qué es un contraejemplo explícito?

He tratado de demostrar la equivalencia anterior, pero me está costando dar una solución con total generalidad. Cualquier ayuda sería apreciada. ¡Gracias!

Editar: Hanul ha demostrado que las condiciones anteriores no son suficientes para establecer la continuidad. ¿Son necesarios?

Respuestas (1)

Podría estar fuera de su alcance, pero su teorema propuesto es falso.

Considere la siguiente función:

h ( norte ) = ( k + 1 ) k si k k norte < ( k + 1 ) k + 1 .
Puedes ver que si k k norte < ( k + 1 ) k + 1 , entonces

  1. h ( norte ) = ( k + 1 ) k = ( 1 + 1 k ) k k k mi norte , y
  2. h ( norte ) = ( k + 1 ) k ( k + 1 ) ( k + 1 ) / 2 norte 1 / 2 .

Considerar A = { ( k + 1 ) k k norte } . Entonces A es pequeño, pero su imagen inversa bajo h es el conjunto de todos los números naturales, entonces h no es continuo.

OK, entonces no es suficiente. ¿Qué hay de necesario?
@RiversMcForge Todavía no tengo idea, aunque creo que se mantiene.
Después de reflexionar sobre su ejemplo (muy inteligente, por cierto), dos pensamientos: 1) Está claro que hay un montón de contraejemplos fáciles para el "teorema propuesto" original que muestran falta de suficiencia, por ejemplo F ( norte ) = k metro para k metro norte < ( k + 1 ) metro y cualquier entero positivo fijo metro 2 . 2) Tal vez un contraejemplo del límite de crecimiento inferior podría venir en forma de alguna función inversa como F ( norte ) = k para k k norte < ( k + 1 ) ( k + 1 ) .